Store #1 has the better price amount , as the price per dozen eggs is $1.44, whereas Store #2 is $1.29 per dozen eggs.
Store #1:
Price per egg = $1.44 / 12 = $0.12
Store #2:
Price per egg = $4.68 / 36 = $0.13
Store #1 has the better price as it is cheaper per egg.
Price comparison:
Store #1: $0.12 per egg
Store #2: $0.13 per egg
Store #1 has the better price, as the price per dozen eggs is $1.44, whereas Store #2 is $1.29 per dozen eggs. To compare the prices, we need to calculate the cost per dozen eggs for each store. To do this, we divide the cost by the number of eggs. For Store #1, the cost per dozen eggs is $1.44 divided by 12, which is $0.12. For Store #2, the cost per dozen eggs is $4.68 divided by 36, which is $0.13. Therefore, Store #1 has the better price, as it is cheaper than Store #2. By comparing prices, we can find the best deal for our money. It is important to compare prices to get the best value for our money, as it can help us save money in the long run. Additionally, by comparing prices, we can make sure we are getting the highest quality product at the best price.
Learn more about amount here
brainly.com/question/28970975
#SPJ4
in parallelogram ABCD, diagonals AC and BD intersect at E. Which statement proves ABCD is a rectangle?
Answer:
AC ≅ BD
Step-by-step explanation:
A parallelogram is a shape that have two pairs of parallel sides with opposite sides having the same length and opposite angles having the same size.
A rectangle is a shape that have two pairs of parallel sides with opposite sides having the same length and each of the four angles is 90°.
A parallelogram is said to be a rectangle if the diagonals of the parallelogram are of the same size (congruent). Therefore in parallelogram ABCD, ABCD is a rectangle if AC is equal to BD. where AC and BD are the diagonals
AC ≅ BD or say Measure of AC = Measure of BD diagonal.
Given that:
ABCD is a parallelogram
AC and BD intersect at E.
A parallelogram is defined as a closed geometric figure that has got 4 straight sides and opposite sides are parallel to each other.
Set of all parallelogram is superset of set of all rectangles.
There is a property about diagonals of a parallelogram that states as;
" A parallelogram is a rectangle if and only if both of its diagonals are equal in size".
Now given parallelogram is ABCD thus its diagonals are AC and BD.
Thus for ABCD to be rectangle we need:
magnitude of AC = magnitude of BC
or
AC ≅ BD
Learn more here:
https://brainly.com/question/1563728
please i'm begging i have so much homework to do i will give 5 stars and mark brainliest
The greatest common factors of the triplets of natural numbers are:
Case 1: G.C.F. = 8
Case 2: G.C.F. = 14
Case 3: G.C.F. = 9
Case 4: G.C.F = 2⁴
Case 5: G.C.F. = 16
Case 6: G.C.F. = 4
Case 7: G.C.F. = 15
Case 8: G.C.F. = 1
Case 9: G.C.F. = 3
Case 10: G.C.F. = 10
Case 11: G.C.F. = 2
Case 12: G.C.F. = 1
Case 13: G.C.F. = 4
Case 14: G.C.F. = 6
Case 15: G.C.F. = 4
Case 16: G.C.F. = 8
How to find the greatest common factor
Natural numbers can be classified into two groups: (i) Prime numbers, (ii) Composite numbers. Composite numbers are products of prime numbers. Greatest common factors are the greatest common natural number that can divide a group on number.
The process to determine the greatest common factor is shown below:
Represent each natural number as a product of prime numbers.Find the common prime numbers of the group of numbers.Obtain the greatest common factors.Case 1
80 = 2⁴ × 5
64 = 2⁶
56 = 2³ × 7
G.C.F. = 2³
G.C.F. = 8
Case 2
56 = 2³ × 7
84 = 2² × 3 × 7
14 = 2 × 7
G.C.F. = 2 × 7
G.C.F. = 14
Case 3
45 = 3² × 5
54 = 2 × 3³
81 = 3⁴
G.C.F. = 3²
G.C.F. = 9
Case 4
64 = 2⁶
16 = 2⁴
32 = 2⁵
G.C.F = 2⁴
Case 5
48 = 3 × 2⁴
64 = 2⁶
96 = 2⁵ × 3
G.C.F. = 2⁴
G.C.F. = 16
Case 6
28 = 2² × 7
48 = 2⁴
96 = 2⁵ × 3
G.C.F. = 2²
G.C.F. = 4
Case 7
90 = 2 × 3² × 5
75 = 3 × 5²
45 = 3² × 5
G.C.F. = 3 × 5
G.C.F. = 15
Case 8
70 = 2 × 5 × 7
54 = 2 × 3²
95 = 5 × 19
G.C.F. = 1
Case 9
33 = 3 × 11
6 = 2 × 3
84 = 2² × 3 × 7
G.C.F. = 3
Case 10
30 = 2 × 3 × 5
90 = 2 × 3² × 5
70 = 2 × 5 × 7
G.C.F. = 2 × 5
G.C.F. = 10
Case 11
100 = 2² × 5²
30 = 2 × 3 × 5
2 = 2
G.C.F. = 2
Case 12
92 = 2² × 23
67 = 67
41 = 41
G.C.F. = 1
Case 13
92 = 2² × 23
16 = 2⁴
20 = 2² × 5
G.C.F. = 2²
G.C.F. = 4
Case 14
54 = 2 × 3³
60 = 2² × 3 × 5
72 = 2³ × 3²
G.C.F. = 2 × 3
G.C.F. = 6
Case 15
12 = 2² × 3
72 = 2³ × 3²
80 = 2⁵ × 5
G.C.F. = 2²
G.C.F. = 4
Case 16
80 = 2⁴ × 5
24 = 2³ × 3
48 = 2⁴ × 3
G.C.F. = 2³
G.C.F. = 8
To learn more on greatest common factors: https://brainly.com/question/11221202
#SPJ1
if your really good with math pls help
Answer:
GIVEN :
Which division expression is equivalent to 4 and one-third divided by Negative Start Fraction 5 over 6 End Fraction?
A) Start Fraction 13 over 3 End Fraction divided by (Negative Start Fraction 5 over 6 End Fraction)
B) Negative Start Fraction 5 over 6 End Fraction divided by Start Fraction 13 over 3 End Fraction
C) Start Fraction 13 over 3 End Fraction divided by Start Fraction 5 over 6 End Fraction
D) Negative Start Fraction 13 over 3 End Fraction divided by (Negative Start Fraction 5 over 6 End Fraction)
TO FIND :
The division expression is equivalent to 4 and one-third divided by Negative Start Fraction 5 over 6 End Fraction
SOLUTION :
Given expression is 4 and one-third divided by Negative Start Fraction 5 over 6 End Fraction.
It can be written as
Now solving the above expression as below :
By converting the mixed fraction into proper fraction,
By using the quotient rule of exponents :
∴ the given expression is equivalent to
⇒ option A) Start Fraction 13 over 3 End Fraction divided by (Negative Start Fraction 5 over 6 End Fraction) is correct.
Step-by-step explanation:
Help please I need help
find X
Answer:
\(x = 145\)
Step-by-step explanation:
1.Approach
To find the value of (x), one will have to take multiple steps. Since a straight line is (180) degrees, one can use this property. Subtract the measure of the exterior angle (the angle formed between the extension of the side of a triangle, and the side of a triangle) from (180) to find an expression to describe the measure of the angle inside the triangle.
Then one is given a regular octagon, using the sum of interior angles formula, find the measure of any interior angle. The vertical angles that when two lines intersect, the opposite angles are congruent. One can use this property to find the final unknown angle in the triangle.
It is known that the sum of angles in a triangle is (180). Use this to find the measure of (x).
2. Find the measure of two angles in the triangle
The degree measure of a straight angle is (180), therefore, when two angles form a straight line, their sum of (180). One can apply this here by stating,
\(x+(unknown)=180\)
Solve for unknown,
\(unknown = 180 - x\)
One can also state,
\(170+(unknown_2)=180\)
Solve for the unknown,
\(unknown_2=10\)
3.Find the degree measure of one of the angles in the octagon
The given octagon (eight-sided figure) is a regular octagon. By its definition, all of the sides in any regular polygon are congruent. This is indicated here, thus the octagon is a regular octagon. One property of a regular polygon is that all of the angles in the figure are cognrunet. This means that the sum of interior angles divided by the number of angles wil give one the measurement of each angle.
The formula to find the sum of interior angles in a polygon is as follows,
\(S=180(n-2)\)
Where (n) is the number of sides. Since an octagon has (8) sides substitute this number into the formula and solve for the sum of angles,
\(S=180(n-2)\\\\S=180(8-2)\)
Simplify,
\(S=180(8-2)\\\\S=180(6)\\\\S= 1080\)
Now divide by the number of angles. The number of angles in an octagon is (8). Since this is a regular octagon, all of the angles are congruent, thus dividing the sum of angles by the number of angles will give on the measure of each angle.
\(1080\) ÷ \(8 = 135\)
4. Find the measure of the final angle in the triangle
The vertical angles theorem states that when two lines intersect, the angles opposite each other are congruent. Therefore, the final unknown angle in this triangle is equal to (135) degrees, because the angle opposite in the octagon is equal to (135) degrees.
5. Find the meausre of (x)
The sum of angles in any triangle is (180) degrees. Since one has found the measure or expression of the angle measure of each angle in a triangle, one can form an equation and solve for the unknown,
\((180-x)+(10)+(135)=180\)
Simplify,
\(325-x=180\)
Inverse operations,
\(325-x=180\\\\-x = -145\\\\x=145\)
What inspires you to make you accomplish your dream job/career
I'm not trying to be nosy just a question hopefully a kind and respectful one
I will give brainest to both ppl who answer and go on there profile and thank them
Answer:
i always think of my future...i want to be a doctor when i grow up and the prospect of growing up to help other people makes me motivated to help myself to reach that goal. Becoming a doctor is really hard work, but just focusing and visualizing my future helps me stay motivated.
for a fourth order runge-kutta approximation of a differential equation. if the time-step used in the approximate is reduced by a factor of 10, by what factor does the error of this approximate get reduced?
If the time step used in the fourth-order Runge-Kutta approximation is reduced by a factor of 10, the error of the approximation is reduced by a factor of 1000.
The error of a fourth-order Runge-Kutta approximation of a differential equation is typically proportional to the time step cubed, assuming the function being approximated is sufficiently smooth.
Therefore, if the time step is reduced by a factor of 10, the error should be reduced by a factor of \(\(10^3\)\), or 1000.
In general, for a fourth-order Runge-Kutta method, if you reduce the time step by a factor of \(\(n\)\), the error is expected to decrease by a factor of \(\(n^3\)\).
Reducing the time step used in a fourth-order Runge-Kutta approximation by a factor of 10 will lead to a reduction in the error of the approximation by a factor of 1000.
Learn more about differential equation here: https://brainly.com/question/33433874
#SPJ11
What are the intercepts for the equation? Select both the x- and y- intercept. 2x + y = 4
Answer:
x=1 y=2
Step-by-step explanation:
Answer:
y= -2x+4
So the y intercept is 4, as its the c in this equation
I think that makes your x intercept as either 8 or 2...
Step-by-step explanation:
572 cars were parked in a parking garage. The same number of cars was parked on each floor. If there were 4 floors, how many cars were parked on each floor? ____ cars
PLEASE HELP FAST 10 POINTS!
Answer:
143 cars on each floor
Step-by-step explanation:
572 / 4 = 143
There were 143 cars on each floor.
What is division?In maths, a division is a process of splitting a specific amount into equal parts.
Given that, 572 cars were parked in a parking garage. The same number of cars was parked on each floor, and there were 4 floors,
In 4 floor = 572 cars
1 floor = 572/4 cars = 143 cars
Hence, There were 143 cars on each floor.
For more references on division, click;
https://brainly.com/question/621288
#SPJ2
Differentiate implicitly to find the first partial derivatives of z.
x+sin(y+z)= 0
The first partial derivatives of z with respect to x and y in the equation x + sin(y + z) = 0 are ∂z/∂x = -1 and ∂z/∂y = -cos(y + z).
To find the first partial derivatives of z with respect to x and y, we need to differentiate the given implicit equation with respect to x and y while treating z as a function of x and y.
Differentiating the equation with respect to x:
∂/∂x (x + sin(y + z)) = 1 + ∂z/∂x
Differentiating the equation with respect to y:
∂/∂y (x + sin(y + z)) = cos(y + z) (1 + ∂z/∂y)
The term ∂z/∂x represents the partial derivative of z with respect to x, and ∂z/∂y represents the partial derivative of z with respect to y.
So, the first partial derivatives of z are:
∂z/∂x = -1
∂z/∂y = -cos(y + z)
These derivatives indicate how the variable z changes with respect to changes in x and y in the given equation x + sin(y + z) = 0. The value of -1 for ∂z/∂x means that for every unit increase in x, z decreases by 1. The value of -cos(y + z) for ∂z/∂y indicates how z changes with respect to changes in y, with the specific relationship determined by the trigonometric function cos(y + z).
Learn more about partial derivatives here:
https://brainly.com/question/32554860
#SPJ11
12.) A jacket cost 4 times as much as a pair of shorts.
Together they cost $75. How much is each item?
Answer:
The pants cost $15 and the jacket costs $60
Step-by-step explanation:
2. The power method covered in Lecture 26 and Section 5.8 relies on the following derivations: Let the eigenvalues 11, ..., An of A be indexed in descending order, so that A1 > 121 > 143 > ... > nl Suppose that the corresponding eigenvectors V1,...,Vn form a basis for R". Let x = civi+...+ CrVn with ci +0. Then A*x= ** (civa +c7 (*) *va + - + en C5) *va). We will explore how the vector (Akx) compare to the eigenvector V1 in magnitude and direction as ko? (a) Let A = et A_ [11 -9 and sol -9 11 an and select the start vector Xo = [ 1 0]. For k = 0,1,...,3, com- pute Xk+1 = (1/4k) Axk, where Hi is the largest entry of Axk. Compare the sequence M1,..., with the largest eigenvalue of A (determined from the roots of the character- istic polynomial) and compare the sequence Xk with the corresponding eigenvector of A (scaled so its largest entry is 1). (b) Repeat part (a), but this time compute Xk+1 = (1/4) A-1Xk, for k = 0,1,...,3, and compare the sequence wi!....Ma with the smallest eigenvalue of A. Connect your observations to your explanation in part (b) by relating the eigenvectors and eigenvalues of A-1 to those of A.
The magnitudes of these vectors decrease as k increases, indicating that the power method converges to the eigenvector V1 = [1 1] in direction. The magnitudes of these vectors also decrease as k increases, indicating convergence to the eigenvector V2 = [1 -1] in direction.
(a) For A = [11 -9; -9 11], the characteristic polynomial is (A - 11)^2 - 81 = 0, which has roots 2 and 20. Thus, the largest eigenvalue of A is 20. The corresponding eigenvector is [1 1] (scaled so its largest entry is 1). Starting with x0 = [1 0], we get the following sequence of vectors: x1 = [0 -0.25], x2 = [0.0625 0], x3 = [0 0.0156].
(b) Using A-1, we have the eigenvalues 1/20 and 1/2, with corresponding eigenvectors [1 -1] and [1 1]. Starting with x0 = [1 0], we get the following sequence of vectors: x1 = [-0.225 0.225], x2 = [0.0506 -0.0506], x3 = [-0.0114 0.0114].
In general, if A has eigenvalues λ1,...,λn with corresponding eigenvectors v1,...,vn, then A-1 has eigenvalues 1/λ1,...,1/λn with corresponding eigenvectors v1,...,vn. The power method applied to A-1 with start vector x converges to the eigenvector corresponding to the smallest eigenvalue of A, while the power method applied to A converges to the eigenvector corresponding to the largest eigenvalue of A.
To know more about magnitudes refer to-
https://brainly.com/question/14452091
#SPJ11
A thief steals an atm card and must randomly guess the correct -digit pin code from a -key keypad. Repetition of digits is allowed. What is the probability of a correct guess on the first try?.
The probability of a correct guess on the first try is 0.024%.
What is probability?
The area of mathematics known as probability deals with numerical representations of the likelihood that an event will occur or that a statement is true.
A number between 0 and 1 is the probability of an event.
where,
P(E) = Number of Favorable Outcomes/Number of Total Outcomes
Because a thief steals an ATM card and must randomly guess the correct four-digit pin code from an eight-key keypad, and digit repetition is permitted, the following calculation must be performed to determine the probability of a correct guess on the first try:
8 \(*\) 8 \(*\) 8 \(*\) 8 = X
\(8 ^ {4 }\) = X
simplifying the above equation, we get
4096 = X
1/4096 = X
0.000244140625 = X
0.000244140625 \(*\) 100 = 0.024%
Therefore, the probability of a correct guess on the first try is 0.024%.
The complete question is;
A thief steals an atm card and must randomly guess the correct four-digit pin code from a 8-key keypad. repetition of digits is allowed. what is the probability of a correct guess on the first try?
To learn more about probability, refer to:
https://brainly.com/question/24756209
#SPJ4
make triangles and find the central angle
Answer:
Step-by-step explanation:
It 14
Answer:
this is like related to fractions right?
Step-by-step explanation:
please help 10 points
Answer:
Sana di ten points
Step-by-step explanation:
B
#carryonlearning
____________ was designed to tabulate the 1890 census and used cards with designated areas representing data fields.
Answer:
Hollerith tabulating machine
Step-by-step explanation:
The Hollerith tabulating machine was invented by Herman Hollerith in other to assist in the data processing of the United States 1890 election. This machine was used to read and summarize the information stored on punchcards. This machine paved the way for the development of enhanced models which were employed for accounting and some other aspects related to business management.
. Which of the following lengths can be the sides of a right-angled triangle? 2cm, 7 cm, 10cm
Step-by-step explanation:
10 cm maybe according to me (*_*)
Adi bought a bag for $25 and sold it at a loss of 10%. Find the selling price of the bag.
Answer:
The selling price of the bag is $22.5
Step-by-step explanation:
Buying Price = $25,
Selling Price = ?
Since they sold it at a loss of 10%,
So, they sold it for a price 10% less than the buying price,
or at 90% or 0.9 of the buying price,
so,
Selling Price = (0.9)(25) = $22.5
A woman has two boxes of matches.she uses five matches and has 75 matches left.how many matches were in each box?
Answer:
40
Step-by-step explanation:
So if you add 5 to 75 you get 80 so that means there were 80 matches all then you divide 80 by 2 and voila 40!
Hoped this helped
P.s. Can I get a happy bday my bday is tmmr
One hundred must be raised to what be equal to a million cubed
Answer:
100^9
Step-by-step explanation:
100 to the 9th power because 100^3 equals 1 million. think of it as (100^3)^3
2x + 4 = 0
find the value of x
Answer:
\(x=-2\)
Step-by-step explanation:
\(2x + 4 = 0\)
Subtract 4 from both sides:-
\(2x+4-4=0-4\)
\(2x=-4\)
Divide both sides by 2:-
\(\frac{2x}{2}=\frac{-4}{2}\)
\(x=-2\)
OAmalOHopeO
\( \huge \boxed{\mathfrak{Question} \downarrow}\)
2x + 4 = 0, value of x = ?\( \large \boxed{\mathfrak{Answer \: with \: Explanation} \downarrow}\)
\( \sf2x+4=0\)
Subtract 4 from both sides. Anything subtracted from zero results in its negation.
\( \sf2x=-4 \)
Divide both sides by 2.
\( \sf \: x=\frac{-4}{2} \\ \)
Divide -4 by 2 to get -2.
\( \boxed{ \boxed{ \bf \: x=-2 }}\)
In triangle QRS, Q = (8x), m/R = (14x+2)°, and m/S= (10x +50)°. What is the measu
A. 4°
B. 32°
C. 58°
D. 90°
Please select the best answer from the choices provided
OA
OB
D
Answer:
C. 58 degrees
To find the measure of angle R in triangle QRS, we can use the fact that the sum of the angles in a triangle is 180 degrees.
Given:
Q = 8x
m/R = 14x + 2 degrees
m/S = 10x + 50 degrees
The sum of angles Q, R, and S is 180 degrees:
Q + R + S = 180
Substituting the given values:
8x + (14x + 2) + (10x + 50) = 180
Simplifying the equation:
8x + 14x + 2 + 10x + 50 = 180
32x + 52 = 180
32x = 180 - 52
32x = 128
x = 128/32
x = 4
Now that we have the value of x, we can substitute it back into the given expressions to find the measures of angles Q, R, and S.
Q = 8x = 8 * 4 = 32 degrees
m/R = 14x + 2 = 14 * 4 + 2 = 58 degrees
m/S = 10x + 50 = 10 * 4 + 50 = 90 degrees
Therefore, the measure of angle R is 58 degrees. So, the correct answer is C. 58°.
I hope you do great and pass this Unit test!!
You are flying a kite in a competition and the length of the string is 725 feet and the angle at which the kite is flying mesures 35” with the ground. How high is your kite flying?
The kite is flying 415.84 feet from the ground
How to determine the height?The given parameters are:
Length of the string, L = 725 feetAngle, x = 35 degreesLet the height be h.
So, we have the following sine function
sin(x) = h/L
Make h the subject
h = L * sin(x)
This gives
h = 725 * sin(35)
Evaluate
h = 415.84
Hence, the kite is flying 415.84 feet from the ground
Read more about elevation at:
https://brainly.com/question/16716174
#SPJ1
Black Diamond Company produces snowboards. Each snowboard requires 2 pounds of carbon fiber. Management reports that 7,000 snowboards and 8,000 pounds of carbon fiber are in inventory at the beginning of the third quarter, and that 170,000 snowboards are budgeted to be sold during the third quarter. Management wants to end the third quarter with 5,500 snowboards and 6,000 pounds of carbon fiber in inventory. Carbon fiber costs $19 per pound. Each snowboard requires 0.5 hour of direct labor at $24 per hour. Variable overhead is budgeted at the rate of $14 per direct labor hour. The company budgets fixed overhead of $1,802,000 for the quarter. Required: 1. Prepare the production budget for the third quarter. Hint: Desired ending inventory units are given. BLACK DIAMOND COMPANY Production Budget (in units) Third Quarter Budgeted sales units Add: Desired ending inventory units Total required units Less: Beginning inventory units Units to produce 170,000 5,500 175,500 7,000 168,500 2. Prepare the direct materials budget for the third quarter. BLACK DIAMOND COMPANY Direct Materials Budget Third Quarter Units to produce Materials needed for production (pounds) Total materials required (pounds) Materials to purchase (pounds) + Cost of direct materials purchases 3. Prepare the direct labor budget for the third quarter. BLACK DIAMOND COMPANY Direct Labor Budget Third Quarter Units to produce Direct labor hours needed Cost of direct labor 4. Prepare the factory overhead budget for the third quarter. BLACK DIAMOND COMPANY Factory Overhead Budget Third Quarter Direct labor hours needed Budgeted variable overhead Budgeted total factory overhead
Production: 168,500 units.
Direct materials: 337,000 pounds.
Direct labor: Calculated based on units produced.
Factory overhead: Calculated based on direct labor hours.
1. Production Budget for the Third Quarter:
BLACK DIAMOND COMPANY Production Budget (in units) Third Quarter
Budgeted sales units: 170,000
Add: Desired ending inventory units: 5,500
Total required units: 175,500
Less: Beginning inventory units: 7,000
Units to produce: 168,500
2. Direct Materials Budget for the Third Quarter:
BLACK DIAMOND COMPANY Direct Materials Budget Third Quarter
Units to produce: 168,500
Materials needed for production (pounds): 2 pounds per snowboard
Total materials required (pounds): 337,000 pounds
Materials to purchase (pounds): Total materials required - Beginning inventory pounds - Desired ending inventory pounds
3. Direct Labor Budget for the Third Quarter:
BLACK DIAMOND COMPANY Direct Labor Budget Third Quarter
Units to produce: 168,500
Direct labor hours needed: 0.5 hour per snowboard
Cost of direct labor: Direct labor hours needed * Direct labor rate
4. Factory Overhead Budget for the Third Quarter:
BLACK DIAMOND COMPANY Factory Overhead Budget Third Quarter
Direct labor hours needed: Calculated from the direct labor budget
Budgeted variable overhead: Direct labor hours needed * Variable overhead rate
Budgeted total factory overhead: Budgeted fixed overhead + Budgeted variable overhead
Note: The specific rates for direct materials, direct labor, and variable overhead were not provided in the given information and would need to be included in the calculations based on the company's specific cost structure.
Learn more about Production here:-
https://brainly.com/question/30284183
#SPJ11
MUST BE IN SPSS program FORMAT NOT WRITTEN OR OTHER SELF MADE GRAPHS PLEASE ONLY SPSS!
An advertising firm wanting to target people with strong desires for success conducted a study to see if such people differed in the types of television shows they watched. Randomly selected participants recorded the shows they watched for a week, then their desire for success was assessed, and finally they were divided into two groups. Low Success seekers watched 8 comedies, 15 romances, 6 documentaries, 13 dramas, and 3 news shows. High Success seekers watched 3 comedies, 3 romances, 9 documentaries, 7 dramas, and 8 news shows. Using this data answer the following:
• State the populations and hypotheses
• Create a table for the data using SPSS
• Conduct a Chi-Squared for independence test using the SPSS program and post output file.
• State the results using the proper APA format.
• Is the distribution of type of shows watched different for participants having high and low desires for success?
Population 1: People with low desire for success.
Population 2: People with high desire for success.
(H₀): The type of television shows watched is independent of the desire for success.
(H₁): The type of television shows watched is dependent on the desire for success.
Population and Hypotheses:
Population 1: People with low desire for success.
Population 2: People with high desire for success.
Null Hypothesis (H₀): The type of television shows watched is independent of the desire for success.
Alternative Hypothesis (H₁): The type of television shows watched is dependent on the desire for success.
Table for the Data :
| | Comedies | Romances | Documentaries | Dramas | News Shows |
|--------------------|----------|----------|---------------|--------|------------|
| Low Success Seekers | 8 | 15 | 6 | 13 | 3 |
| High Success Seekers | 3 | 3 | 9 | 7 | 8 |
Learn more about Hypothesis here:
https://brainly.com/question/32562440
#SPJ1
Solve this problem for 100 points
The graph of each function in the piecewise function is added as an attachment
How to determine the plot of the piecewise function?The equation of the piecewise function is given as
f(n) = n/2 if n is even
3n + 1 if n is odd
In the above definition of the piecewise function, we have the following functions and the domains
f(n) = n/2 at a domain of even numbersf(n) = 3n + 1 at a domain of even numbersNote that:
The domain of the function is the set of input values of the graph
This in other words mean the domain of a function is the set of x values of the graph.
Next, we plot the graph of each function in the piecewise function in their respective domain
See attachment for the graph of each function in the piecewise function,
On the graph, the blue line represents f(n) = n/2 while the other line represents f(n) = 3n + 1
Read more about piecewise function at
brainly.com/question/3628123
#SPJ1
❗️❗️URGENT❗️❗️
3. Match the graph with the correct equation.
A. y-1=-6/5(x-2)
B. y+2=6/5(x+1)
C. y-2=6/5(x-1)
D. y+1=6/5(x+2)
\(3 \times 4x \times 2y\)
how do you simplify?
Answer:
24 x y
Step-by-step explanation:
Simplify the following:
3×4×2 x y
Hint: | Multiply 3 and 4 together.
3×4 = 12:
12×2 x y
Hint: | Multiply 12 and 2 together.
12×2 = 24:
Answer: 24 x y
Write an expression that represents "the sum of x and 8".
Answer: x+8
The term 'sum' means 'result of adding two (or more) numbers or expressions'.
If a wheelchair-marathon racer moving at 13.1 miles per hour expends energy at a rate of 645 calories per hour, how much energy in calories would be required to complete a marathon race (26.2 miles) at this pace
Answer:
\(1290\) calories
Step-by-step explanation:
Given: The racer moves at \(13.1\) miles per hour expends energy at a rate of \(645\) calories per hour.
To find: Energy in calories, required to complete a marathon race \(26.2\) miles at this pace.
Solution: We have,
The racer moves at \(13.1\) miles per hour.
The racer expends energy at a rate of \(645\) calories per hour.
So, energy expended while moving \(13.1\) miles \(=645\) calories.
Now, energy expended while moving \(1\) mile \(=\frac{645}{13.1}\) calories.
So, energy expended while moving \(26.2\) miles \(=\frac{645}{13.1}\times 26.2=645\times 2=1290\) calories.
Hence, \(1290\) calories of energy is required to complete a marathon race \(26.2\) miles at this pace.
The wheelchair-marathon racer would need 1290 calories to complete the marathon race.
SpeedSpeed is the ratio of distance travelled to total time taken. It is given by:
Speed = distance / time
The speed of the racer is 13.1 miles per hour. To complete a 26.2 miles:
13.1 = 26.2 / t
t = 2 hours.
The racer expends energy at a rate of 645 calories per hour, for 2 hours:
Amount of calories = 645 calories per hour * 2 hours = 1290 calories
The wheelchair-marathon racer would need 1290 calories to complete the marathon race.
Find out more on speed at: https://brainly.com/question/4931057
Factorise 2x^2 + x - 6
\( \rm \: {2x}^{2} + x - 6\)
\( \rm \: {2x}^{2} + 4x - 3x - 6\)
\( \rm \: {2x} \times (x + 2)- 3x \times (x + 2)\)
\( \rm \: (x + 2) \times (2x - 3)\)